LSAT and Law School Admissions Forum

Get expert LSAT preparation and law school admissions advice from PowerScore Test Preparation.

User avatar
 Dave Killoran
PowerScore Staff
  • PowerScore Staff
  • Posts: 5853
  • Joined: Mar 25, 2011
|
#44164
Complete Question Explanation
(The complete setup for this game can be found here: lsat/viewtopic.php?t=9437)

The correct answer choice is (C)

Answer choice (A) is proven incorrect by the following hypothetical: J J J J.

Answer choice (B) is proven incorrect by the following hypothetical: L L L L.

Answer choice (C) is the correct answer. If M is selected, then O must be selected, and if O is selected, then two Os must be selected. Also, if O is selected then P must be selected. So, at a minimum, if M is selected, then at least four fish must be selected.

Answer choice (D) is proven incorrect by the following hypothetical: O O P.

Answer choice (E) proven incorrect by the following hypothetical: P O O.

Please note that answer choices (A) and (B) are functionally identical since they both hinge on a random. Thus, answer choices (A) and (B) are both incorrect. A similar, but slightly different relationship exists between answer choices (D) and (E). The same hypothetical eliminates each answer choice, so once you determine one answer choice is incorrect, you can eliminate the other answer choice.

Get the most out of your LSAT Prep Plus subscription.

Analyze and track your performance with our Testing and Analytics Package.